25
$\开始组$

整数序列在线百科全书中的序列A0000607是将$n$划分为素数部分的分区数。例如,有$10$的$5$分区为素数部分:$10=2+2+2+2+2=2=2+2+3=2+3+3+3+3+5=3+7=5+5

$$A000607(n)\sim\exp\left(2\pi\sqrt{\frac{n}{3\logn}}\right)$$

从数值上看,即使你取两边的对数,这似乎也是错误的。我的猜测是

$$\lim_{n\to\infty}\log\left(A000607(n)\right)\bigg/\left$$

请参见下图:

比率对数图

如何证明或反驳这一推测?

有关更多参考,请参阅http://oeis.org/A000607.

$\端组$
2

1答案1

重置为默认值
42
$\开始组$

您的数据与Vaughan在Ramanujan J.15(2008),109–121中证明的更精确的估计值相一致。他的定理1和2(以及他的(1.9))揭示了$$\log(A000607(n))=2\pi\sqrt{\frac{n}{3\logn}}\左(1+\frac}\logn{\lognneneneep{\logn}+O\左(\frac{1}{\log n}\右)\右)$$对于$n=50000$,我们有$$\log(A000607(n))\约252.663$$$$2\pi\sqrt{\frac{n}{3\logn}}\约246.601$$$$2\pi\sqrt{\frac{n}{3\logn}}\左(1+\frac{\log\n}{\logn{right)\约300.877$$因此,如果你使用沃恩公式中的二次项,近似值(没有误差项)不会低于实际值,而是高于实际值。我们还看到,在这个特定的例子中,误差为$\约48.214$,这与上述误差项为$O(1)$倍的事实非常吻合$$2\pi\sqrt{\frac{n}{3\logn}}\cdot\frac{1}{\logn{约为22.792$$

简而言之,你的猜测可能是错误的,而沃恩是正确的。数值异常是由一个二次项引起的,该二次项对于您考虑的$n$而言相当大。

$\端组$
8
  • 1
    $\开始组$ 非常感谢,我承认我的猜想是错误的,小渐近项很重要。 $\端组$ 评论 2014年9月15日8:39
  • 1
    $\开始组$ 我没有仔细检查这份文件,所以我不能肯定,但是arxiv.org/pdf/1609.06497.pdf声称Vaughan的二阶项中有一个错误,它是$-\frac12\frac{\log\logn}{\logn{$,并且$\logn$项上的常数在这里被标识。至少,物理论文的答案更符合数学。 $\端组$
    – 露西亚
    评论 2017年5月24日3:03
  • $\开始组$ @露西亚:我认为物理学家是对的!在我看来,沃恩在第118页的第二次显示中犯了一个简单的计算错误。$\log\log x$的系数应该是$+1/2$,而不是$1$。因此,在本页随后的显示中,$\log\log x/\log x$的所有系数都应该乘以$-1/2$,这就纠正了定理1,使其与物理学家的结果相一致。你能确认一下吗?如果你同意,那么我会更新上面的回复,并向所有作者发送一条消息(同时提及你)。 $\端组$ 评论 2017年5月24日16:32
  • $\开始组$ 嗨GH:我目前无法访问沃恩的论文,过几天我会看一看。我相信你是对的,这是一些简单的计算错误。 $\端组$
    – 露西亚
    评论 2017年5月25日1:52
  • $\开始组$ @露西亚:如果有帮助的话,我可以通过电子邮件发送这份文件。基本上,Vaughan看的是一个函数$X(X)$,使得$X(X)^2/\log X(X”)$是渐近的$(6/\pi^2)X$。然后,他推导出$\log X(X)$等于$\frac{1}{2}\log X-\log X+O(1)$,而不是$\frac{1}{2}\log X+\frac{1}}{2\log X+O(1。 $\端组$ 评论 2017年5月25日2:19

你的答案

单击“发布您的答案”,表示您同意我们的服务条款并确认您已阅读我们的隐私政策.

不是你想要的答案吗?浏览已标记的其他问题问你自己的问题.